Pagina 1 di 1

$1,\ldots,n$ coperti da progr geom- parte 3

Inviato: 22 ott 2013, 20:47
da jordan
Own. Mostrare che se $k$ progressioni geometriche coprono i numeri $1,2,\ldots,n$ quando $n$ รจ sufficientemente grande, allora $k>n/4$.